5
$\begingroup$

start of proof

That's what I have so far... It seems like a bad approach. I've tried others and end up in the same spot.

$\endgroup$
3
  • 1
    $\begingroup$ @Danikar: using a truth-table is not the method that the OP needs to use to prove the equivalence. $\endgroup$
    – amWhy
    Jan 16, 2013 at 23:02
  • $\begingroup$ I don't mean to be a bother, but what textbook is this particular problem from? $\endgroup$
    – Mack
    Feb 13, 2013 at 22:16
  • 1
    $\begingroup$ @Eli it's this one amazon.com/gp/aw/d/0073383090 $\endgroup$
    – papercuts
    Feb 14, 2013 at 1:46

3 Answers 3

9
$\begingroup$

What you've done in the first few steps is all correct. I'll start where you left off. We need to use a lot of distribution. And it gets messier before it gets clearer!

To prove $$[(p \rightarrow q) \land (q\rightarrow r)] \rightarrow (p\rightarrow r) \equiv T$$ $$\vdots$$ $$\vdots$$

$$\equiv (p\land \lnot q) \lor (q \land \lnot r) \lor (\lnot p \lor r)\tag{picking up...}$$ $$\equiv [[(p\land \lnot q) \lor q] \land [(p \land \lnot q) \lor \lnot r]] \lor (\lnot p \lor r)\quad \tag{distributivity x 2}$$

$$\equiv [[(p\lor q) \land (\lnot q \lor q)] \land [(p\lor \lnot r)\land (\lnot q \lor \lnot r)]] \lor(\lnot p \lor r)\quad \quad\quad \quad\tag{distributivity x 2}$$

$$\equiv [(p \lor q) \land T \land (p \lor \lnot r) \land (\lnot q \lor \lnot r)] \lor (\lnot p \lor r)\quad \tag{$\lnot q \lor q \equiv T$}$$

$$\equiv [(p \lor q) \land (p \lor \lnot r) \land (\lnot q \lor \lnot r)] \lor (\lnot p \lor r)\quad \quad \quad \quad \tag{$(p \lor q) \land T \equiv (p \lor q)$} $$


Can you see how distribution (as discussed in an answer to your earlier question) helps here? We've eliminated one expression ($\lnot q \lor q \equiv T$), and if you proceed with expanding out, using distribution, on the expression to the left (in brackets), you will be able to eliminate other terms...ending with a final evaluation of $T$.

Why don't you work on it a bit to see what you arrive at, and post a follow up question in a comment below, or as an edit to this question if you run into problems.


Edit: continued from where we left off...

To prove:

$$[(p \rightarrow q) \land (q\rightarrow r)] \rightarrow (p\rightarrow r) \equiv T$$ $$\vdots$$ $$\vdots$$

$$\equiv [(p \lor q) \land (p \lor \lnot r) \land (\lnot q \lor \lnot r)] \lor (\lnot p \lor r)\quad \quad \quad \quad \tag{$(p \lor q) \land T \equiv (p \lor q)$} $$ $$\equiv [(p \lor q) \land (\lnot q \lor \lnot r) \land (p\lor \lnot r)] \lor (\lnot p \lor r)\quad\quad\quad\quad \tag{Commutative property}$$

$$\equiv[[(p \lor q) \land \lnot q] \lor [(p \lor q) \land \lnot r] \land (p \lor \lnot r)] \lor (\lnot p \lor r)\quad\quad\quad\quad \tag{distributivity x 2}$$

$$\equiv[[(p\land \lnot q) \lor (q \land \lnot q) \lor (p \land \lnot r) \lor (q \land \lnot r)] \land (p \land \lnot r)] \lor (\lnot p \lor r) \quad\quad\quad\quad\quad \tag{distributivity}$$

$$\equiv [[(p \land \lnot q) \lor F \lor (p\land \lnot r) \lor (q \land \lnot r)] \land (p \land \lnot r)] \lor (\lnot p \lor r)\quad\quad\quad\quad\quad\quad\tag{$q \land \lnot q \equiv F$}$$

$$\equiv [[(p \land \lnot q) \lor \color{red}{\bf{(p\land \lnot r)}}\lor (q \land \lnot r)] \land \color{red}{\bf{(p \land \lnot r)}}] \lor (\lnot p \lor r) \quad\quad\quad\quad\quad\quad\quad\quad\quad \quad\quad\quad \tag{$(p \land \lnot r) \lor F \equiv p \land \lnot r)$}$$

$$\equiv \color{red}{\bf{(p \land \lnot r)}} \lor (\lnot p \lor r)\quad\quad\quad\quad \tag{?}$$

$$\equiv (\lnot \lnot p \land \lnot r) \lor (\lnot p \lor r)\quad\quad\quad\quad\tag{Double negation}$$

$$\equiv \lnot( \lnot p \lor r) \lor (\lnot p \lor r) \quad\quad\quad \tag{DeMorgan's Law}$$

$$\quad \equiv T \quad\quad\quad\quad \tag{$\lnot a \lor a \equiv T$}$$

Hence, $$[(p \rightarrow q) \land (q\rightarrow r)] \rightarrow (p\rightarrow r) \equiv T$$


Task: What remains is for you to justify/understand why the step followed by "$\,(?)\,$" holds.


$\endgroup$
10
  • $\begingroup$ i.imgur.com/8exCR.png It's just getting worse... $\endgroup$
    – papercuts
    Jan 16, 2013 at 4:08
  • 1
    $\begingroup$ I'll take a look at it...I got to a F (false) "or" term, and dropped it as well...but I'm going to need to get a little sleep. I'll look at it more carefully when I wake up. I think we can trim back a little on the work...but I'm proud of your effort! I'll get back to you! $\endgroup$
    – amWhy
    Jan 16, 2013 at 4:23
  • $\begingroup$ Thanks, you're awesome. Applying the distributive across 4 terms helped me with a different problem as well so double thanks ! $\endgroup$
    – papercuts
    Jan 16, 2013 at 4:38
  • 1
    $\begingroup$ @amWhy, your proof strategy is crazy! The formula $[(p \rightarrow q) \land (q\rightarrow r)] \rightarrow (p\rightarrow r)$ is true in a positive implicational calculus --- you do not have to introduce these negations, conjunctions and use the law of excluded middle. In fact the formula corresponds to the modus ponens rule, or to internal "composition": $\endgroup$ Jan 17, 2013 at 14:30
  • 1
    $\begingroup$ @amWhy - I don't agree. Papercuts proposed an approach and wrote "It seems like a bad approach". The correct answer to his question is "yes, it is a bad approach. Here is a (possible) right one.". What you have proposed is CRAZY! $\endgroup$ Jan 17, 2013 at 18:03
0
$\begingroup$

Let's assume that (1)$[(p \rightarrow q) \land (q \rightarrow r)] \rightarrow (p \rightarrow r)$ is false for some assignment of $p$, $q$, and $r$. From the truth table for implication, this means that (2)$(p \rightarrow q) \land (q \rightarrow r)$ must be true while (3)$p \rightarrow r$ is false. From (2) we see that (4)$p \rightarrow q$ and (5)$(q \rightarrow r)$ must be true and from (3) we have (6)$p$ must be true while (7)$r$ is false. So from (4) and (6), $q$ must be true. But now we have a contradiction with (5) and (7). Therefore no assigment of $p$, $q$, and $r$ which will make the original statement false.

$\endgroup$
0
$\begingroup$

Here is yet another proof: \begin{align} & ((p \rightarrow q) \land (q \rightarrow r)) \rightarrow (p \rightarrow r) \\ \equiv & \;\;\;\;\;\text{"expand all occurrences of $\;\rightarrow\;$"} \\ & \lnot((\lnot p \lor q) \land (\lnot q \lor r)) \lor \lnot p \lor r \\ \equiv & \;\;\;\;\;\text{"in the first disjunct we may assume the negation of the others ($\;\lnot p\;$ and $\;r\;$)"} \\ & \lnot((\text{false} \lor q) \land (\lnot q \lor \text{false})) \lor \lnot p \lor r \\ \equiv & \;\;\;\;\;\text{"simplify"} \\ & \lnot(q \land \lnot q) \lor \lnot p \lor r \\ \equiv & \;\;\;\;\;\text{"excluded middle"} \\ & \lnot \text{false} \lor \lnot p \lor r \\ \equiv & \;\;\;\;\;\text{"simplify"} \\ & \text{true} \\ \end{align}

$\endgroup$

You must log in to answer this question.

Not the answer you're looking for? Browse other questions tagged .